Determine whether a quadratic model exists for the set of values below. If​ so, write the model. f(0) = -4, f(3) = -19, f(-1) = -11

Select the correct choice below​:

A) A quadratic model does exist. It is ​f(x)= ?
B) A quadratic model does not exist.

Answers

Answer 1

Answer:

  A) The model exists: f(x) = -3x^2 +4x -4

Step-by-step explanation:

A quadratic model will always exist for 3 given points, provided they are not on a line. In that case, a linear model is appropriate.

Here, the slope between -1 and 0 is positive, and the slope between 0 and 3 is negative. Thus, we know these points are not collinear, and a model must exist.

The model is most easily found using a quadratic regression tool. Such is shown in the attachment. It tells us that ...

  f(x) = -3x^2 +4x -4

Determine Whether A Quadratic Model Exists For The Set Of Values Below. If So, Write The Model. F(0)

Related Questions

Evaluate this function: h(t) = |t+2| + 3; Find h(6)

Answers

Answer:

Step-by-step explanation:

h(6) = |6 + 2| + 3 = |8| + 3 = 8 + 3 = 11

The chart shows how many people have signed up to go on a field trip each day. 62 students are allowed to go on the field trip. On which day would you expect that number to be reached?A) 8B) 9C) 10D) 11

Answers

Answer:

C

I hope it helps.

Solve the equation: 3.017 + k = 5.134

Answers

Answer:

k=2.117

Step-by-step explanation:

Answer:

3.017 + k = 5.134

So, k = 5.134 - 3.017

So, k = 2.117

which number will reach if we move 6 steps to the right -3

Answers

Answer:

If you meant -3 + 6, then the correct answer is 3.

You should put some extra effort in composing questions with all the required information, like a picture maybe.

Answer:

your answer has to be 3

Step-by-step explanation:

if you were to move to the right in number line you have to add so -3+6 is equals to 3

8y + 15x + 3y - 12x - 3x

Answers

Answer:

11y is the exact value.

Step-by-step explanation:

geomtry plz help 15 points

Answers

Answer:

it should be 60°. the single marks on each side means they are all equal

Answer:

m∠L=60º

Step-by-step explanation:

The triangle is an equilateral triangle meaning that all angles and sides are equal.

m∠L=60º

Hope this helps :)

Two trains leave towns 474 kilometers apart at the same time and travel toward each other. One train travels13 km/m faster than the other. If they meet in 2 hours, what is the rate of each train?

Answers

Answer:

112 km/h

125 km/hr

Step-by-step explanation:

Distance = time * speed

Distance  = distance train 1 + distance train 2

474 = 2(s) + 2(s + 13)

Distribute

474 = 2s + 2s + 26

Combine like terms

474 = 4s + 26

Subtract 26 from both sides

448 = 4s

Divide both sides by 4

112 = s

s = 112 km/h

s + 13 = 125 km/hr

geomtry plzzz help 15 points

Answers

Answer:

19

Step-by-step explanation:

they give you xz, and they want half, so just divide 38 by 2

What is 75.65 rounded to the nearest whole number?​

Answers

It is 76 as the other person said, what I like to think to help me remember is five or more raise the score, 4 or less, let it rest

solve the following stystem -4x + 2y = 0 and x + y = -1

Answers

The (x, y) values are (-1/3, -2/3) respectively

Question 2 Fill-In-The-Blank Worth 3 points)
(04.04)
Point R is at (4.1.5) and Point T is at (429) on a coordinate grid. The distance between the two points is_(Input numbers and decimal point only, such as 82)
Answer for Blank 1

Answers

Answer:

1.4

Step-by-step explanation:

in the graph, the points are 14 points apart, when the step of the graph is set to .1, if you count, they're .14 point apart, or 1.4

if u help me ill give u brainliest​

Answers

Answer:

150000000

Step-by-step explanation:

30x5000000=150000000

in a supermarket, credit card purchases have an additional charge of 8% of the purchase made. If I bought 2 suitcases that cost s / .165.60 each and paid with a credit card, how much do I have to pay?

Answers

9514 1404 393

Answer:

  357.70

Step-by-step explanation:

The final cost will be ...

  (number of suitcases) × (cost per suitcase) × (1 +8%)

  = 2(165.60)(1.08) = 357.70

The width of a triangle is six more than twice the height. The area of the triangle is 88in2. Find the height and width of the triangle.

Answers

Answer:

w = 22 in, h = 8 in.

Step-by-step explanation:

Reading the problem, I immediately see two equations. This hints me towards a system of equations problem.

Statement 1 is The width of a triangle is six more than twice the height.

The width (w) of a triangle is (=) six (6) more (+) than twice (2) the height (h).

Equation 1: w = 6 + 2h

Statement 2 is The area of the triangle is 88 in2.

The formula for the area of a triangle is [tex]\frac{wh}{2}[/tex]. So, we get our second equation.

Equation 2: wh/2 = 88

We see that we have the same number of equations as unknowns, two. This is important as it lets us know that we can find two unknowns in the two equations. If there are more unknowns than equations, then we cannot find the values of all the variables. Look for another equation in the problem. However, if the # equations greater than or equal to number of unknowns, then we are ready to start solving!  So, we can now move to solving.

Equation 1 isolates one of the unknowns (w). So, we can replace w in the second equation with the right side of eq 1. Then, we simplify for h.

[tex]\frac{wh}{2} = 88[/tex]

[tex]\frac{(6 + 2h)h}{2} = 88[/tex]

[tex]\frac{6h + 2h^2}{2} = 88[/tex]

[tex]6h + 2h^2 = 176[/tex]

[tex]2h^2 + 6h - 176 = 0[/tex]

[tex]h = -11, 8[/tex] (assuming you know how to solve quadratics)

Since we know that heights of triangles cannot be negative, we find that [tex]h = 8[/tex] in.

For the final step, we plug [tex]h[/tex] into our first equation to find [tex]w[/tex].

[tex]w = 6 + 2h[/tex]

[tex]w = 6 + 2(8)[/tex]

[tex]w = 6 + 16[/tex]

[tex]w = 22[/tex]

So, the answer is w = 22 in, h = 8 in.

To check if we were right, we can plug the values into both equations to see if they are equal. I will skip equation 1 since we used that equation "as-is" to find w.

[tex]\frac{wh}{2} = 88[/tex]

[tex]\frac{22 * 8}{2} = 88[/tex]

[tex]11 * 8 = 88[/tex]

[tex]88 = 88[/tex]

The solution checks out!

e/kaid_274404147475337871026587/assig!
Khan Academy
Percent word problems
Of the 50 U.S. states, 4 have names that start with the letter W.
What percentage of U.S. states have names that start with the letter W?
0
Stuck? Review related articles/videos or use a hint.

Answers

Answer:

Step-by-step explanation:

4/50 = 0.08

0.08(100) = 8%

does the graph represent a function?

Answers

Answer:

No

Step-by-step explanation:

fails the vertical line test, a test to see if a vertical line can touch 2 points on a graph at the same time.

Match the event with the most appropriate probability word: The roll of a normal fair dice will show a 7.

A. Certainly
B. Likely
C. Evens
D. Unlikely
E. Impossible
F. Other

Answer this, plz.

Answers

Answer:

Option 3

Evens

. . . . . .. . ..

Last year at a certain high school, there were 100 boys on the honor roll and 60 girls on the honor roll. This year, the number of boys on the honor roll decreased by 11% and the number of girls on the honor roll decreased by 10%. By what percentage did the total number of students on the honor roll increase? Round your answer to the nearest tenth (if necessary).​

Answers

Answer:

-11 boys -6 girls

Step-by-step explanation:

2x+5y=(-24),3x+y=(-10) solve for using elimination method

Answers

Answer:

x equals -2 and y equals−4

g A long-term study has revealed that a test for cancer in men is very effective. The study shows that 89% of the men for which the test is positive actually have cancer. If a man selected at random tests positive for cancer with this test, what is the probability that he does not have cancer

Answers

The answer is 89% because I had that Same question and it was 89%

What’s the slope of 10,6 and 4, 1.2

Answers

let's firstly change the 1.2 to a fraction

[tex]1.\underline{2}\implies \cfrac{12}{1\underline{0}}\implies \cfrac{6}{5} \\\\[-0.35em] ~\dotfill\\\\ (\stackrel{x_1}{10}~,~\stackrel{y_1}{6})\qquad (\stackrel{x_2}{4}~,~\stackrel{y_2}{\frac{6}{5}}) \\\\\\ \stackrel{slope}{m}\implies \cfrac{\stackrel{rise} {\stackrel{y_2}{\frac{6}{5}}-\stackrel{y1}{6}}}{\underset{run} {\underset{x_2}{4}-\underset{x_1}{10}}}\implies \cfrac{~~ \frac{6-30}{5}~~}{-6}\implies \cfrac{~~ \frac{-24}{5}~~}{-6}\implies \cfrac{~~ -\frac{24}{5}~~}{-\frac{6}{1}}[/tex]

[tex]-\cfrac{\stackrel{4}{~~\begin{matrix} 24 \\[-0.7em]\cline{1-1}\\[-5pt]\end{matrix}~~}}{5}\cdot -\cfrac{1}{\underset{1}{~~\begin{matrix} 6 \\[-0.7em]\cline{1-1}\\[-5pt]\end{matrix}~~}}\implies \boxed{\cfrac{4}{5}}[/tex]

if a chocolate cake is divided into six equal slices and each slice has 280 calories

Answers

Answer:

Whats the question?

Step-by-step explanation:

Answer:

1,680

Step-by-step explanation

6 slices each 280 calories, 280 x 6 = 1,680

Juan wants to buy a video game for $63. He saves $12 every Friday. Part A. Create an equation to represent Juan's total savings, y, in dollars, after x Fridays. y​

Answers

Answer:

Answer in explanation.

Step-by-step explanation:

An equation to represent Juan's total savings y, in dollars, after x Fridays will be;

⇒ y = 12x

What is an expression?

Mathematical expression is defined as the collection of the numbers variables and functions by using operations like addition, subtraction, multiplication, and division.

Given that;

Juan wants to buy a video game for $63.

And, He saves $12 every Friday.

Now,

Let total dollars after x Friday = y

And, He saves $12 every Friday.

So, We can formulate;

⇒ Total money (y) = 12x

⇒ y = 12x

Thus, An equation to represent Juan's total savings y, in dollars, after x Fridays will be;

⇒ y = 12x

Learn more about the mathematical expression visit:

brainly.com/question/1859113

#SPJ2

A breakfast cereal producer makes its most popular product by combining just raisins and flakes in each box of cereal. The amounts of flakes in the boxes of this cereal are normally distributed with a mean of 370\,\text{g}370g370, start text, g, end text and a standard deviation of 24\,\text{g}24g24, start text, g, end text. The amounts of raisins are also normally distributed with a mean of 170\,\text{g}170g170, start text, g, end text and a standard deviation of 7\,\text{g}7g7, start text, g, end text.
Let T=T=T, equals the total amount of product in a randomly selected box, and assume that the amounts of flakes and raisins are independent of each other.
Find the probability that the total amount of product is less than 575\,\text{g}575g575, start text, g, end text.
You may round your answer to two decimal places.
P(T<575)\approx

Answers

Using the normal distribution, it is found that there is a 0.9192 = 91.92% probability that the total amount of product is less than 575 g.

In a normal distribution with mean [tex]\mu[/tex] and standard deviation [tex]\sigma[/tex], the z-score of a measure X is given by:

[tex]Z = \frac{X - \mu}{\sigma}[/tex]

It measures how many standard deviations the measure is from the mean.  After finding the z-score, we look at the z-score table and find the p-value associated with this z-score, which is the percentile of X.When two normal variables are added, the mean is the sum of the means while the standard deviation is the square root of the sum of the variances.

In this problem, the product is composed by flakes and raisins, and we have that:

[tex]\mu_F = 370, \sigma_F = 24, \mu_R = 170, \sigma_R = 7[/tex]

Hence, the distribution for the total amount of product has mean and standard deviation given by:

[tex]\mu = \mu_F + \mu_R = 370 + 170 = 540[/tex]

[tex]\sigma = \sqrt{\sigma_F^2 + \sigma_R^2} = \sqrt{24^2 + 7^2} = 25[/tex]

The probability that the total amount of product is less than 575 g is the p-value of Z when X = 575, hence:

[tex]Z = \frac{X - \mu}{\sigma}[/tex]

[tex]Z = \frac{575 - 540}{25}[/tex]

[tex]Z = 1.4[/tex]

[tex]Z = 1.4[/tex] has a p-value of 0.9192.

0.9192 = 91.92% probability that the total amount of product is less than 575 g.

A similar problem is given at https://brainly.com/question/22934264

what is 4% of 32? is it a percent, base, or amount

Answers

Answer:

1.28 - amount

Step-by-step explanation:

You would multiply 32 times 0.04 and get 1.28 which I assume would be an amount.

The amount of 4% of 32 is 1.28.

We have to determine what is 4% of 32.

What is the percentage?

This free percentage calculator computes a number of values involving percentages, including the percentage difference between two given values.

Suppose it is an x percent.

Therefore x multiply 32 times 0.04 and get 1.28 an amount.

To learn more about the percentage of visits:

https://brainly.com/question/24304697

#SPJ2

the odds against your bet are to 3. if you bet $27 and win. how much you win​

Answers

If you win, you will get eighty-one dollars ($81.00)
If you win you will get $81 because 3*27= 81

If you paid $160 in interest to a loan company for the use of $10000 of or 30 days, what annual simple interest rate did they charge?

Answers

Answer:A loan company charges $30 interest for a one month loan of $500. Find the annual interest rate they are charging.

Step-by-step explanation:You will earn $160 interest total over the four years. Try it Now 1. A loan company charges $30 interest for a one month loan of $500. Find the annual interest ...

What is the coefficient of x in expression of 5mx​

Answers

Coefficient of this expression is 5 only.

- BRAINLIEST answerer

2x - 5y = 6 and -2x + 7y = 14

*Using elimination*​

Answers

2x-5y+ (-2x) + 7y= 6+14
2y = 20
y= 10
2x-5(10)=6
2x= 56
x= 28
here is the ans.hope this helps!

Jason gets 15 gallons of gas for $3.90 a gallon which gives him 20 miles to the gallon. How many miles can Jason travel on one tank of Jason?

Answers

0.75 miles
15 divided by 20 as its in proportion

Answer:

0.75 mi.

Step-by-step explanation:

15/20=3.90/x

Other Questions
Competition was fierce. People spoke of nothing else in the days that _________, admiring the winner and promising themselves that one day, their own kite would fly proudly over the town. Please help with this other question PLEASE HELP ME: With which statement would the authors of Congressional Reconstruction MOST LIKELY agree? A. The Confederates are traitors who left the Union and should not be readmitted without caution and oversight. B. The former Confederates are still Americans and should be granted a full pardon if they take an oath of allegiance. C. We should abolish slavery, but we should not upset the social order by granting full citizenship rights to African Americans. D. Although we must protect the rights of newly freed African Americans, state governments should be restored to their prewar state. The school store sells Midville High T-shirts for $12 each and Midville High gym shorts for $9 a pair. One afternoon they sold a total of 53 T-shirts and pairs of shorts for a total of $570 . How many T-shirts did they sell and how many pairs of shorts did they sell? A man is sitting on a chair with wheels. He grabs a 2.1 kg book from the desk and throwsthe book at a speed of 7.2 m/s. His chair moves backward. The man has a mass of 70 kg andthe chair has a mass of 9.2 kg. What is the speed of the man and the chair after the book isthrown? can someone give me answers plzz Order the numbers 0.5, 1.25, 0.75, and 1.08 from least to greatest. What is the difference of the polynomials? (9x2 +8x) -(2x2+3x) 7x2+5x 7x2+11x 11x2+5x 11x2+11x Which tab must be enabled to access the VBA Editor?O DeveloperO InsertO ReviewO View what is the right hotspot please help T or F1. Plains are never found along coastlines 9Describe the actions of Linda's father on the ice. Ln(x)+ln(x+7)=1solve this log equation please help me answer these three question 20 points1) Some southerners argued that slavery was actually GOOD for many slaves. What was their reasoning?2) Why were families and religion important to many enslaved people?3) Name one subtle or understated form of resistance that many enslaved people used. 8. What is the relationship between multiplying and factoring? YOU SHOULD BE USING THE IMPERFECT Define Explicit Cost .[tex] \: \: [/tex] Mr. and Mrs. Bruce took a boat trip which left Cala for Friends Island at 11:45 in the morning. The trip lasted for 1 hour and 20 mins. When did they arrive at Friends Island? ng A trc khi cht c lm di chc cho v 100 triu, cn li chia u cho cc con v chu. Hy chia ti tn ca ng A?- TH1; Di chc ming khng hp php-TH2; Di chc ming hp phpBit ti sn ca ng A l 1 t cc con u thnh nin how do you use fossils to see change